What is the value of the expression?
8 1/2-2+4 3/4

Answers

Answer 1
11 1/4 or 11.25 is the answer

Related Questions

sacar el perimetro de esta figura

Answers

Answer: 6x^2 + 10x -2

Añade todos los lados de la figura y combina los términos iguales por ejemplo todos los que tienen “x” en frente, súmales

please can you help me thanks

Answers

Answer:

Step-by-step explanation:

mode means most repeated number . here in this given data 7 is repeated maximum number of times . so mode is 7.

to find median arrange the data from ascending to descending order.

Data = 1,2,3,4,5,7,7,7,8,8

median = N + 1/2    (N means no of data )

=10 + 1/2

=11/2

=5.5

= 5th term + 6th term/2

=5+7/2

=12/2

=6

mean = sum of data / no of data

=3+7+2+4+7+5+7+1+8+8/10

=52/10

=5.2

range = largest value - smallest value

=8 - 1

=7

PLEASE HELP ASAP!
a. 4
b. 8
c. 16
d. 24
which one????

Answers

Answer:

A,B,C,D,E,F,G,H,I,J,K,L,M,N,O,P,Q,R,S,T,U,V,W,X,Y,Z

the answer is a : 4

3. Escribe la ecuación de la elipse que cumple con:
F1(-3,0) F2(3,0) y a=4

Answers

Answer:

90

Step-by-step explanation:

Question 2 of 25
If you apply the changes below to the absolute value parent function, f(x) = [XI,
what is the equation of the new function?
• Shift 4 units to the right.
• Shift 6 units up.

Answers

Answer:

[tex]g(x) = |x-4|+6[/tex]

Step-by-step explanation:

shift 4 to the right --> x-4

shift 6 up --> +6

Kelsey is solving the inequality 17+4<9 17 + 4 x < 9 . To solve the inequality, she begins by solving the equation as shown. Which number line represents the solution to the inequality? 17 + 4 x = 9. Subtract 17 from both sides. 4 x = negative 8. Divide both sides by 4. X = negative 2. CLEAR CHECK A number line going from negative 10 to positive 10. A closed circle is at negative 2. A number line going from negative 10 to positive 10. Closed circles are at negative 4, negative 3, negative 2, negative 1, 0. A number line going from negative 10 to positive 10. An open circle is at negative 2. The line is shaded to the right of the circle. A number line going from negative 10 to positive 10. An open circle is at negative 2. The line is shaded to the left of the circle.

Answers

Answer:

an open circle will be at -2(since the inequality sign does not include "equal to") and the line is drawn to the left of the circle (since the sign is "left than")

Step-by-step explanation:

Given the inequalities

17+4x < 9

Subtract 17 from both sides

17+4x -19 < 9 - 17

4x < -8

Divide both sides by 4

4x/4 < -8/4

x < -2

To represent the solution on a number line, an open circle will be at -2(since the inequality sign does not include "equal to") and the line is drawn to the left of the circle.

The formula for the volume of a cylinder is Y = gr). Solve y = grn for h, the height of the cylinder. O A. = O B. has Yr2 OC. h - TV O D. h =


PLEASE HELP ASAPPPPPPPPPPPPP​

Answers

since we have to solve the given formula for h(height) so, the correct option is (a)

helpppp i neeeed to know this in order to pass

Answers

B

The y-values go from at least -6 to at most 9 so B fits that

Answer:

-6 <=y <=9

Step-by-step explanation:

simplify this expression - 1/2 ( -5/6 + 1/3 )

Answers

5/6 divided by 1/3 multiplied by 1/2
You cannot divide, you have to multiply the reciprocal of the 2nd fraction, it would turn into:
5/6 multiplied by 3/1 multiplied by 1/4
Next step, set up the problem:
%285%2F6%29%283%2F1%29%281%2F4%29
Now multiply across:
numerator:
5 x 3 = 15
15 x 1 = 15
denominator:
6 x 1 = 6
6 x 4 = 24
15%2F24
Simplify, 3 is the highest number able to be divided into both:
15/3 = 5
24/3 = 8
5%2F8

A recipe uses 3 cups of milk to make 9 servings. If the same amount of milk is used for each serving, how many servings can be made from two quarts?
1 gallon
=
1 gallon=


4 quarts
4 quarts
1 quart
=
1 quart=


2 pints
2 pints
1 pint
=
1 pint=


2 cups
2 cups
1 cup
=
1 cup=


8 fluid ounces
8 fluid ounces
Before you try that problem, answer the question below.
How many cups will you need to find the number of servings for?

Answers

Answer:

24 servngs

Step-by-step explanation:

3 cups makes 9 servings.

The rate of servings to cups is 9 servings to 3 cups which is a unit rate of 3 servings per cup.

1 quart = 4 cups

2 quarts = 2 * 4 quarts = 8 cups

3 serviongs/cup * 8 cups = 24 servings

Answer: 24 servings

a triangle has coordinates a(-1,-2) b(-4,-2) and c(-4,-5) what are the coordinates of a' the image of point a under a dilation with a scale factor of 3?

Answers

9514 1404 393

Answer:

  (-3, -6)

Step-by-step explanation:

The scale factor multiplies each coordinate.

  A' = 3A = 3(-1, -2) = (-3, -6)

The image of A is ...

  A'(-3, -6)

PLEASE ANSWER ASAP IM TAKING FINALS
sara is painting her house. to paint the second floor, she needs to lean her 15 foot ladder against her house to reach a point that is 11 feet above the ground

A) What is the angle of elevation that Sara's ladder will form with the ground?
B) How far from the base of the house should Sarah place the base of her ladder?

Answers

Answer:

A) 47.17

B) 10.2 feet

Step-by-step explanation:

dont forget to switch the calculator to degrees when doing this

For A)

arcsin(11/15)=angle of elevation

47.17=angle of elevation

For B)

tan(47.17)=11/x

11/tan(47.17)=x

10.2=x

read comments for other stuff that i forgot to put in here

Angle of elevation is equals to 47.14 degrees and distance from the base of the house is equals to [tex]2\sqrt{26} foot[/tex].

What is angle of elevation?

" Angle of elevation is defined as the angle between horizontal plane and the observation line of sight."

Formula used

[tex]sin\alpha = \frac{opposite side}{hypotenuse}[/tex]

Pythagoras theorem

[tex](Hypotenuse)^{2} = (Perpendicular)^{2} + (Base)^{2}[/tex]

According to the question,

Length of the ladder ' hypotenuse'= 15 foot

Height to reach a point ' perpendicular' = 11feet

'[tex]\alpha[/tex]' represents the angle of elevation

[tex]'x'[/tex] represents the base of the house

Substitute the value to get the value of angle of elevation we have,

   [tex]sin\alpha = \frac{11}{15}[/tex]

⇒ [tex]\alpha = sin^{-1} (0.7333)[/tex]

[tex]\alpha = 47.14[/tex] degrees

Substitute the value in the formula to get the value of base we have,

[tex](15)^{2}= (11)^{2} +x^{2}[/tex]

⇒[tex]x^{2} = 225 - 121[/tex]

⇒[tex]x^{2} = 104[/tex]

⇒[tex]x=\sqrt{104}[/tex]

[tex]x= 2\sqrt{26}[/tex] foot

Hence, angle of elevation is equals to 47.14 degrees and distance from the base of the house is equals to [tex]2\sqrt{26} foot[/tex].

Learn more about angle of elevation here

https://brainly.com/question/21137209

#SPJ2

A turkey has a mass of 6 kilograms. A bag of sweet potatoes has a mass of 2 kilograms. How many more grams does the turkey weigh than the sweet potatoes?

Answers

Answer:

the correct answer is 4,000 grams

The diagram shows the length and width of a cell phone, and the length of a larger
version of the same brand of cell phone.
2.6 in.
5.4 in.
6.21 in.
The lengths and widths of the two cell phones are proportional. What is the width,
in inches, of the larger version of the cell phone?
A
1.15
B
2.26
С
c
2.99
D
3.41

Answers

Answer: all the above
Explanation: because

Will give BRAINLIEST to the correct answer.

Use a calculator. Round to the nearest tenth of a degree.

Given cos 0 = 0.9326, find 0.

Answers

The answer:
0=cos^-1 (0.9326)
0=21.156

Nearest tenth:
0=21.2

Solve for the class width if R = 40 and I = 7.

A. 5
B. 6
C. 8
D. 7

Answers

Answer:

A.5

Step-by-step explanation:

divide 40 by 7 which is equal to 5

write the equation of a line with that passes through given point and gas the given slope. Answer in general form​

Answers

Step-by-step explanation:

c) since m = undefined => the equation is

x = -2

d) the equation is

y-2= -1/7 (x+7)

=> y = -1/7 x -1+2

<=> y = -1/7 x + 1

The length of a rectangle is 5m longer than the width. The perimeter of the rectangle is 82m. Find the length and the width of the rectangle. *

Answers

Answer:

+ 5 = length 2x + 2(x + 5) = 38

Take the square of every integer between $-20$ and $20$ (including $-20$ and $20$), and write them down on a list. The first few numbers in the list should be $(-20)^2 = 400$, $(-19)^2 = 361$, and $(-18)^2 = 324$. How many different numbers are there on the list

Answers

Answer:

There are 21 distinct numbers on the list

Step-by-step explanation:

Square of integers between - 20 and 20

-20² = 400

-19² = 361

-18² = 324

-17² = 289

-16² = 256

-15² = 225

-14² = 196

-13² = 169

-12² = 144

-11² = 121

-10² = 100

-9² = 81

-8² = 64

-7² = 49

-6² = 36

-5² = 25

-4² = 16

-3² = 9

-2² = 4

-1² = 1

0² = 0

1² = 1

2² = 4

3² = 9

4² = 16

5² = 25

6² = 36

7² = 49

8² = 64

9² = 81

10² = 100

11² = 121

12² = 144

13² = 169

14² = 196

15² = 225

16² = 256

17² = 289

18² = 324

19² = 361

20² = 400

Help its in the picture

Answers

Answer:

$190

Step-by-step explanation:

My knowledge!

Hope it helps!

Answer:

190

Step-by-step explanation:

Step 1 find total amount of money she has saved

To do this simply multiply the amount of money she saves per week ( 4.75 ) by the number of weeks ( 8 )

Hence money she has saved = 8 * 4.75 = 35

Step 2 subtract cost of camera by amount of money she has saved

228 - 38 = 190

So we can conclude that she needs 190 more dollars to purchase the camera.

Please help me it's urgent!

Answers

Answer:

y = 2x-5

y=-x+4

Step-by-step explanation:

Just need to solve the:

x+y=4

y-2x=-5

And we have:

x=3

y=1

Then replace x and y into the answer to find the correct one

can you please please help me.

Answers

Answer:

yes

Step-by-step explanation:

yes

Answer:

[tex]x=22\text{ and } y=123[/tex]

Step-by-step explanation:

By definition, the interior angles of a triangle must sum to 180°. Therefore:

[tex](2x+13)+(57)+(3x)=180[/tex]

Combine like terms:

[tex]5x+70=180[/tex]

Subtract 70 from both sides:

[tex]5x=110[/tex]

And divide both sides by five. Hence:

[tex]x=22[/tex]

Next, notice that y and the 57° angle form a linear pair. By definition, linear pairs equal 180°. Therefore:

[tex]y+57=180[/tex]

So:

[tex]y=123[/tex]

Alternatively, we can use the Exterior Angle Theorem, which states that the exterior angle is equivalent to the sum of the two interior angles opposite to the exterior angle. In other words:

[tex]y=(2x+13)+(3x)[/tex]

Since we've determined that x = 22:

[tex]y=(2(22)+13)+(3(22))=44+13+66= 123[/tex]

We acquire the same answer.


What is the exact area of shaded region of the circle? Show all work.

Answers

Answer:

[tex]A=\frac{513\pi}{8}[/tex]

Step-by-step explanation:

1.Approach

To solve this problem, find the total area of the circle. Then multiply the total area by the fraction of the circle that is shaded.

2.Find the area of the circle

The formula to find the area of a circle is the following,

[tex]A=(\pi)(r^2)[/tex]

Where ([tex]r[/tex]) represents the radius of the circle, that is the distance from the center of the circle to the circumference of the circle (the outer edge). ([tex]\pi[/tex]) represents the numerical value (3.1415..).

Substitute the given values into the formula and solve for the area.

[tex]A=(\pi)(r^2)\\\\A=(\pi)(9^2)\\\\A=(\pi)(81)[/tex]

3.Find the percent of the circle that is shaded

There are a total of (360) degrees in a circle. It is given that (75) degrees are not shaded. To find the fraction of a circle that is shaded, subtract (75) form (360). Then put this value over (360) and simplify.

[tex]360 - 75 = 285[/tex]

Now put this number over (360) to find the fraction of the circle that is shaded.

[tex]\frac{285}{360}[/tex]

[tex]=\frac{19}{24}[/tex]

4.Find the area of the shaded region

Now multiply the total area by the fraction that the shaded region occupies.

[tex](81)(\pi)*(\frac{19}{24})\\\\=\frac{(81)(19)(\pi)}{24}\\\\=\frac{513(\pi)}{8}[/tex]

what is the greatest 5 digit number which when divided by 2, 3, 4, and 5 leaves a remainder of 1 in each case​

Answers

Answer:99904.

Step-by-step explanation:

Find the LCM of

5

10 = 2x5

15 = 3x5

20 = 2x2x5

25 = 5x5

LCM = 2x2x3x5x5 = 300

Take the smallest 5-digit number: 10000 and divide it by 300 to get 33.33. Round it off to 34 and multiply it by 300 to get 10200. Finally add 4 to 10200 to get 10204 which is the smallest final 5-digit number.

Check: 10204/5 = 2040 as quotient and a remainder of 4. Correct.

10204/10 = 1020 as quotient and a remainder of 4. Correct.

10204/15 = 680 as quotient and a remainder of 4. Correct.

10204/20 = 510 as quotient and a remainder of 4. Correct.

10204/25 = 408 as quotient and a remainder of 4. Correct.

Answer: 10204.

To get the greatest 5-digit number take 99999 and divide it by 300 to get 333.33. Round it off to 333 and multiply it by 300 to get 99900. Finally add 4 to 99900 to get 99904 which is the final greatest 5-digit number.

Check: 99904/5 = 19980 as quotient and a remainder of 4. Correct.

99904/10 = 9990 as quotient and a remainder of 4. Correct.

99904/15 = 6660 as quotient and a remainder of 4. Correct.

99904/20 = 4995 as quotient and a remainder of 4. Correct.

99904/25 = 3996 as quotient and a remainder of 4. Correct.

Answer: 99904.

Mr. Baker is driving 100 miles. His average speed, to the nearest 2 miles per hour, is 56 miles per hour. What is the range of possible values

for t, the total time of the drive in hours?

A

1.79
B

1.72 < t < 1.85

с

1.72
1.75 < t < 1.82

Answers

Answer:

1.72 ≤ t ≤ 1.85

Step-by-step explanation:

Given :

Distance = 100 miles ;

Speed = 56 miles per hour, to the nearest 2 miles per hour

Hence, the speed range is :

(56 - 2) mph ; (56 + 2) mph

Range of possible Time taken, t :

Distance / speed ;

100 / 58 ≤ t ≤ 100 / 54

1.72 ≤ t ≤ 1.85

. Solve for the missing side in the right triangle below.​

Answers

x = √29

Step-by-step explanation:

By the Pythagorean theorem,

A² + B² = C²

14² + x² = 15²

x² = 15² - 14²

x² = 29

x = √29

Step-by-step explanation:

A^2+B^2=C^2

14^2+x^2=15^2

196 + X^2=225

X^2=225-196

x^2=29

X=√29

HELP ASAP!!! I WILL GIVE YOU A brainliest and a 5 star if you give me the correct answer for my homework!!!!! DO NOT WING IT TO GET POINTS FROM ME

Question 1
A bag contains 1 red, 2 blue, 4 orange, and 3 purple marbles. A marble is drawn and not replaced. Then another marble is drawn.

Part A: What is P (purple, then orange)?

Question 2: Use the photo that I attach!!!

Answers

Answer:

Question 1

probability of a red marble in the first draw = 1/10

probability of a orange marble in the second draw = 4/9

probability of getting red in first and orange in second (without replacement) = 1/10 * 4/9 = 2/45,

Question 2

6 outcomes

1/6

Round your answer to the nearest hundredth.
А
(20
?
B
С
3
( need answers plz )

Answers

Answer:

8.24243226

Step-by-step explanation:

Same as the previous one I answered.

Tan = Opposite/ Adjacent

Adjacent = Opposite / Tan(20)

Help please
9. Is 23.32 = 65? Explain.
Type here
TS

Answers

Answer:

No, because 2^3 × 3^2 is equivalent to 72 which is a much smaller number than 6^5 which is equal to 7,776.

Step-by-step explanation:

2^3 × 3^2 = 6^5 ??

Use PEMDAS:

(2 × 2 × 2) × (3 × 3) = 6 × 6 × 6 × 6 × 6

(4 × 2) × 9 = 36 × 6 × 6 × 6

8 × 9 = 216 × 6 × 6

72 = 1,296 × 6

72 = 7,776

Step-by-step explanation:

no because they just added they exponents together. 6^5 is like saying 6 multiplied by itself 5 different times. same with the other numbers. if you switch the exponents around and multiply at different times it's like multiplying different numbers, because 2^3 is multiplying 2 by itself 3 individual times. if you move the exponent to the 6, you are multiplying a completely different number by its own self, so the two different methods cannot be congruent.

(I'm sorry I hope that made sense)

also you can just check with the calculator, this is just an explanation on why it will show up like that on the calculator

What is the size of the matrix resulting from

Answers

Answer:

The answer is 1 * 3.

Step-by-step explanation:

To find the size of the resulting matrix, start by finding the dimensions of both matrices. For the dimensions of a matrix, it goes rows by columns. To find the dimensions of a matrix, count the number of rows, and then count the numbers of columns.

For the first matrix, the dimensions are 1 by 3. For the second matrix, the dimensions are 3 by 3.

An easy way to see if matrices that are being multiplied produce a real answer, find the dimensions of each matrix and see if the two inner numbers match.

For these matrices being multiplied, the dimensions are 1 by 3 and 3 by 3, and the two inner numbers are 3, which means they match.

Then, begin solving for the answer resulting from the matrices being multiplied. Start by multiplying each row in the first matrix by each column in the second matrix. Next, simplify each element of the matrix by multiplying out all the expressions. Finally, the matrix in the most simplified form is [tex]\left[\begin{array}{ccc}11&11&-2\\\end{array}\right][/tex]. Thus, resulting in a 1 * 3 matrix.

Other Questions
There are 5,000 books in the town's library. Of these, 4,700 are fiction. To find the percent of the books that are fiction, first set up the percent equation. Then find the percent. Which operation should you perform first in the expression 7x2^3? Evaluate the expression for the given value of the variable.3x + 4 when x = 25y - 3.6 when y = 47z4+ 1.05 when z = 1.220 - b3.2when b = 5.2 Help please and thank you Can someone help me with this question? In Fahrenheit 451, what would be a theme that could be used in a literary essay? give a 20-30 words explanation on offline school There are _____ countries in Western Europe. Davis's Snacks will make 8,412 ounces of corn chips next year. The company plans to put the chips into 4-ounce bags. How many bags will the company be able to fill next year? PLEASE HELP ASAP Determine the period. How are dates and times stored by Excel? Use working as a noun in a sentence Why are walking, yoga and water exercises so popular as lifetime sports?O Easy on the jointsO Simple to learnO No equipment neededO All of the answer choices How many miles did it cruise? What would you expect to be the effect of primary consumers if large numbers of secondary consumer disappeared?An increase due to loss of predatorsA change in the main diet of primary consumersA decrease due to loss of productionNo change, the system would immediately rebalance 06.04 find the area of the image below a.24.6 units b.25.8 units c.26.3 units d.27.5 units SCIENCE SUBJECT1. Elma Muros Posadas was one of the country's best Elma Muros Posadas was one of the country best long jumpers. Suppose she makes a 30 angle with the ground before leaping. If her launch speed is 11m/s, compute the time of flight, the range and the maximum height reached.Given:__________Required:_______Solution:Final Answer:_______( Nonsense will be reported! ) All of the following items may be sent via email EXCEPT *1 pointsoft copy attachmentshard copy attachmentsweb linksweb pages Complete the proof that what is the domain of the function represented by the graph